LSAT and Law School Admissions Forum

Get expert LSAT preparation and law school admissions advice from PowerScore Test Preparation.

 PeterC123
  • Posts: 26
  • Joined: Dec 27, 2016
|
#31617
Hi,

I have a problem with answer B and D.

The reason why I choose B instead of D is in the passage they said the tenants of rent control units have the power to enact and repeal the rent control ordinances, so if they have this power and all they wanted was short term gain, wouldn't it make more sense for them to get the short term gain of lower rent increases and then just repeal it periodically from time to time so they don't get the long term negative effects?

As I'm writing this I'm starting to see why I'm wrong....is my reasoning wrong b/c they don't want to repeal it b/c as soon as they repeal it the landlords can just jack up prices? So they will hold on to the rent control ordinances for a long time?

My problem with D is that in the passage they never talked about the proportion of people with rent control, what if there's only 0.001 percent of renters living in rent control units, then it probably will not make a difference. Or does the percentage not matter b/c of the word "eventually" in the answer choice?

Tbanks,
 David Boyle
PowerScore Staff
  • PowerScore Staff
  • Posts: 836
  • Joined: Jun 07, 2013
|
#31625
PeterC123 wrote:Hi,

I have a problem with answer B and D.

The reason why I choose B instead of D is in the passage they said the tenants of rent control units have the power to enact and repeal the rent control ordinances, so if they have this power and all they wanted was short term gain, wouldn't it make more sense for them to get the short term gain of lower rent increases and then just repeal it periodically from time to time so they don't get the long term negative effects?

As I'm writing this I'm starting to see why I'm wrong....is my reasoning wrong b/c they don't want to repeal it b/c as soon as they repeal it the landlords can just jack up prices? So they will hold on to the rent control ordinances for a long time?

My problem with D is that in the passage they never talked about the proportion of people with rent control, what if there's only 0.001 percent of renters living in rent control units, then it probably will not make a difference. Or does the percentage not matter b/c of the word "eventually" in the answer choice?

Tbanks,

Hello PeterC123,

"In many municipalities, specifically in all those where tenants of rent-control units have a secure hold on political power and can get rent-control ordinances enacted or repealed" tells us that many municipalities have rent control. Therefore, there'll eventually be rental shortages, as per answer D.
Answer B is much more speculative. Rent control might be repealed if there are shortages, or it might not. We don't know. ...You hypothesize that renters might want to repeal the ordinances from time to time. But, many of them do want low rent; and we don't know if it would work successfully to repeal the ordinances. (E.g., maybe after 5 years of rent control, that will automatically cause housing shortages at some point, so that repealing rent control for a little while isn't going to help rental shortages any.)

Hope this helps,
David
 Mi Kal
  • Posts: 48
  • Joined: Jun 10, 2017
|
#37283
Hi,

I don't understand why D is the answer. D discusses the fact that "there is now, or eventually will be, a shortage," but we don't know that. It sounds like pure speculation. Let's say that there are rent control buildings in a really bad area, a part of town that no one wants to live in. This area has all kinds of violence, drugs, theft, and on top of that the apartments are small, dingy, and in a state of disrepair. The area and the apartments are so awful that the landlords are even offering several months of free rent on top of the rent control. All these incentives still don't mean that the apartments will get rented and that there will be a shortage.

On the other hand I think C makes a lot more sense. I can't see how C can be wrong. I would think that with the financial upside for renters (which is a downside for landlords) that a shortage in available apartments would be the only downside for renters who want those rent controlled units. How can C be wrong?

Thanks.

Michael
 Luke Haqq
PowerScore Staff
  • PowerScore Staff
  • Posts: 751
  • Joined: Apr 26, 2012
|
#37642
Hi Michael,

Regarding answer (D) in this must-be-true question, that answer choice states:
  • (D) In many municipalities there is now, or eventually will be, a shortage of rental units.
We know this must be true because we are told (1) "In many municipalities, specifically in all those where tenants of rent-control units have a secure hold on political power and can get rent-control ordinances enacted or repealed, it is invariably the desire for short-term gain that guides those tenants in the exercise of that power"; and (2) "rent-control ordinances...have several negative effects for renters. One of these is that the controls will bring about a shortage of rental units." In other words, there exist municipalities that have rent control, and those municipalities therefore are experiencing or will experience the negative effect of a shortage of rental units, which is what (D) gets across.

Answer (C) states,
  • (C) The only negative effect of rent control for renters is that it brings about a shortage of rental units.
As I read it, (C) must actually be false. We're given statement (2) above ("rent-control ordinances...have several negative effects for renters. One of these is that the controls will bring about a shortage of rental units."). Given (2), it can't be the case that shortage of rental units is the only negative effect of rent control ordinances.
 Mi Kal
  • Posts: 48
  • Joined: Jun 10, 2017
|
#37737
Thanks Luke.

I see it now "only" v "several."

Michael
 tvaughanPS
  • Posts: 2
  • Joined: Aug 04, 2017
|
#37993
I am still failing to see why D is correct. I think It is because of the term many. Not all municipalities are rent control. So how can many of them have a shortage of units.
 nicholaspavic
PowerScore Staff
  • PowerScore Staff
  • Posts: 271
  • Joined: Jun 12, 2017
|
#38031
Hi tvaughn,
Good question. Here this Must Be True question is not only testing your understanding of the term "many." In fact, like other MBT questions, the language is fluid and you have to consider the whole sentence in order to get all the ramifications of what the author is arguing. So let's reconsider this statement with some emphasis added:

In many municipalities, specifically in all those where tenants of rent-control units have a secure hold on political power and can get rent-control ordinances enacted or repealed, it is invariably the desire for short-term gain that guides those tenants in the exercise of that power.
Here, the author is narrowing down the "many" municipalites to those where the tenants have power and can exercise it. But how will they exercise it? They will "invariably" choose "short term gains" (i.e. immediate, small rent increases). Thus, Answer (D) "In many municipalities there is now, or eventually will be, a shortage of rental units" becomes an inevitable outcome of tenants choosing short term gains over long term disadvantages. That's why it is correct and why "many" ultimately turns into "all."

I hope this helps! :-D
 Tony_Stark
  • Posts: 15
  • Joined: Jun 29, 2017
|
#38091
I have a question about E.

How is it that in the long term rents will not go up? Is it because of the use of "substantially?"

Because from the stimuli, they say "regulations placing limits on rent increases" meaning that rent will go up, just slowly. And over a long period of time, this could be a substantial increase.

Thanks in advance.
 AthenaDalton
PowerScore Staff
  • PowerScore Staff
  • Posts: 296
  • Joined: May 02, 2017
|
#38368
Hi Tony,

You're on the right track by focusing in on the word "substantially." :)

We're told in the stimulus that with rent controls in place, rents will increase by smaller amounts and / or be limited -- if anything, tenants will probably avoid substantial rent increases. Since this is a must be true question, we really need direct support from the stimulus's argument to support an answer choice. The stimulus tells us that in the long run there will be a shortage of units, but we just don't have enough from the stimulus to conclude that rents will increase substantially in the long run.

I hope that makes sense. Good luck studying!

Athena Dalton
 kelia16
  • Posts: 3
  • Joined: Nov 02, 2017
|
#41130
Can someone explain me why E is not the answer ?

Get the most out of your LSAT Prep Plus subscription.

Analyze and track your performance with our Testing and Analytics Package.